Convert 60 degrees to Radians.

Answers

Answer 1
The answer is 30 that what I think

Related Questions

3/4x + 12 =48 what is the value of ' x '​

Answers

Answer:

3/4x + 12 =48

3/4x=36

x=48

Hi, does anyone know the answer to this question? I’m bad at geometry and I’m struggling to answer it.

Answers

The answer is (2) a translation

Please Mark BRAINLIEST!

A wire attached to the top of a pole reaches a stake in the ground 20 feet from the foot of the pole and makes an angle of 58° with the ground. Find the length of the wire.

Answers

it is a right triangle
cos 58 = 20/length of rope
length of rope = 20/cos 58 degrees
= 37.74 feet

HELPP PLS ASAP I NEED HELP

Answers

Answer:

Option (b) second one is the correct answer

3 / 1/3 dividend by 1 / 1/5​

Answers

The answer for this question is 0.2

factorise the following (a+b)⁴-(a-b)⁴​

Answers

Answer:

hope it is helpful to you

this is another answer

sorry for bad handwriting

actually its my sista handwriting

Question
2x + 3y = 11
x – 3y =1
What is the solution (x, y) to the given system of equations?
O (4,0)
O (4,1)
O (12,8)
O (12, 10)

Answers

Answer: (4, 1)

Step-by-step explanation:

Let's solve this by adding the equations

2x+3y=11

+x-3y=1

3x=12

Now we can solve

[tex]\frac{3x}{3} =\frac{12}{3} \\x=4[/tex]

Now that we know x=4 it can only be the first to options. Let's sub in the value of x to one of the original equations to find the value of y.

[tex]2(4)-3y=11\\8+3y=11\\8+3y-8=11-8\\\\3y=3\\\frac{3y}{3} =\frac{3}{3} \\y=1[/tex]

The answer is (4, 1)

If cos theta = 0.8, find 1 / sin (pi/2 - theta)

Answers

Answer:

J

Step-by-step explanation:

Using the cofunction identity

cosθ = sin([tex]\frac{\pi }{2}[/tex] - θ )

Then

[tex]\frac{1}{sin(\frac{\pi }{2}-0) }[/tex]

= [tex]\frac{1}{cos0}[/tex]

= [tex]\frac{1}{0.8}[/tex]

= 1.25 → J

The value of 1/sin([tex]\frac{\pi }{2}[/tex] - θ) is 1.25.

What are four quadrants of trigonometry?

The coordinate axes divide the plane into four quadrants, labelled first, second, third and fourth as shown. Angles in the third quadrant, for example, lie between 180 degrees and 270 degrees.

Given

1/cosθ = 0.8

1/sin([tex]\frac{\pi }{2}[/tex] - θ) =?

By using quadrants in trigonometry

we know that sin([tex]\frac{\pi }{2}[/tex] - θ) = cosθ

= 1/sin([tex]\frac{\pi }{2}[/tex] - θ)

= 1/cosθ

= 1/0.8

= 1.25

1/sin([tex]\frac{\pi }{2}[/tex] - θ) = 1.25

Hence, the value of 1/sin([tex]\frac{\pi }{2}[/tex] - θ) is 1.25.

Learn more about four quadrants of trigonometry here

https://brainly.com/question/21864197

#SPJ2

2) Simplify: 4/3+3/5+-3/5+-11/3​

Answers

Answer:

[tex]\frac{4}{3}+\frac{3}{5}+-\frac{3}{5}+-\frac{11}{3}[/tex][tex]=\frac{4}{3}-\frac{11}{3}[/tex][tex]\frac{a}{c}\pm \frac{b}{c}=\frac{a\pm \:b}{c}[/tex][tex]=\frac{4-11}{3}[/tex][tex]\:4-11=-7[/tex][tex]=\frac{-7}{3}[/tex][tex]\:fraction\:rule}:\quad \frac{-a}{b}=-\frac{a}{b}[/tex][tex]\frac{-a}{b}=-\frac{a}{b}[/tex][tex]\frac{-7}{3}[/tex][tex]=-\frac{7}{3}[/tex]---------------------[tex]\boxed{\boxed{\underline{\textsf{\textbf{hope it helps}}}}}[/tex][tex]\boxed{\boxed{\underline{\textsf{\textbf{have a great day!}}}}}[/tex]

A circle has a radius of 18 ft and an arc length of 117. What is the measure of the central angle of
the circle?

Answers

20akakdjhajaksakakka

what is the solution to the system of equations graphed below?​

Answers

I believe it would be (A) 0.-4

two rectangles have a scale factor of 4:1. If the area of the larger rectangle is 156cm^2, what is the area of the smaller rectangle?

A. 26.75 cm^2
B. 31 cm^2
C. 9.75 cm^2
D. 39 cm^2

Answers

Answer:

c=9.75cm^2

Step-by-step explanation:

156/16 because you are deivding each side by 4 and 4x4=16 and the area is 156

What is the 30th term of the linear sequence below?
-4,-1,2,5,8

Answers

Answer:

83

Step-by-step explanation:

a_1 = -4

a_2 = -1

a_2 - a1 = -1 - (-4) = 3

Each new term is 3 added to the previous term.

a_n = a_1 + 3(n - 1)

Test: Let's find the 4th term and see if we get a_4 = 5.

a_4 = a_1 + 3(4 - 1) = -4 + 3(3) = -4 + 9 = 5

We see that the 4th term really is equal to 5.

a_30 = a_1 + 3(30 - 1) = -4 + 3(29) = -4 + 87 = 83

[tex]P=\frac{11}{40}+\frac{1}{4}-\frac{1}{20}[/tex]

Answers

Answer

p = 19/40

Please give brainliest

which of the two rational numbers 3/5 and -2/3 is greater

Answers

3/5 is greater than -2/3

Which is equivalent to
1/4x

Answers

Answer:

1/4x is basically x divided by 4/multiplied by 1/4

this can be applied in examples such as:

1/4(2) = 0.5

etc.

Step-by-step explanation:

i hope this helped, if not, please leave a comment specifically requiring what exactly you are stuck with.

Answer:

 1/8x+1/8x

Step-by-step explanation:

Hi, does anyone know the answer to this question? I’m bad at geometry and I’m struggling to answer it.

Answers

Answer:

QT = 16

Step-by-step explanation:

ΔQRS ~ ΔQRT

In similar triangles, corresponding angles are in same ratio.

[tex]\frac{QS}{QR}=\frac{QR}{QT}\\\\\frac{25}{20}=\frac{20}{QT}[/tex]

Cross multiply,

QT * 25 = 20 * 20

[tex]QT =\frac{20*20}{25}\\\\QT = 4*4\\\\QT = 16[/tex]

if you know the answer please help me ese ​

Answers

Answer:

Domain { 4,8,12,16,20}

Range { 1,2,3,4}

This is not a function because two of the inputs have more than one output

Step-by-step explanation:

The domain is the x values ( input)

Domain { 4,8,12,16,20}

The range is the y values (output)

Range { 1,2,3,4}

This is not a function because two of the inputs have more than one output

can some one pls help me with this i need help

Answers

Answer:

9/8 cup peanuts were used

The amounts are:

Correct

Correct

Incorrect

Marc is building a rectangular wooden frame for his canvas. If the canvas is 7 feet long by 5 feet wide, what is the perimeter of the canvas?

Answers

Answer:

24 feet

Step-by-step explanation:

Use the perimeter formula, P = 2l + 2w, where l is the length and w is the width.

Plug in the values, and solve:

P = 2l + 2w

P = 2(7) + 2(5)

P = 14 + 10

P = 24

So, the perimeter of the canvas is 24 feet

Homework June 10th 2021 Jim

Answers

Problem 1

Part (a)

It is possible to find the equation of the line of best fit by hand, but it's much more efficient to use technology. That could mean a graphing calculator, an online tool, or some computer software installed. I'm going to use GeoGebra. Specifically, I'm using the "FitLine" command to find the regression line.

The equation of line of best fit is y = -2.45x + 11.83

There's not much to say in terms of steps, since it's basically a calculator problem. Doing this by hand would take a lot longer than it should be.

------------

Part (b)

Plug in x = 2.3 and evaluate

y = -2.45x + 11.83

y = -2.45*2.3 + 11.83

y = 6.195

Answer: 6.195

-------------

Part (c)

We'll compare the result from the previous part (6.195) to the y value in the table (6.2)

The difference is 6.2 - 6.195 = 0.005

This means the estimated value is off by 0.005 and this is an underestimate (since 6.195 is smaller than 6.2)

===================================================

Problem 2

To be perfectly honest, I'm not sure what's going on here. It seems like there's missing context to the problem. Perhaps a data table that got cut off or this is referring to a previous problem.

===================================================

Problem 3

Recall that y = mx+b is the slope intercept form

m = slope

b = y intercept

We simply read off the values of the given equation y = 0.404x - 5.18 to see that 0.404 is the slope and -5.18 is the y intercept

The slope tells us that each time x goes up by 1, y will increase by an estimated amount of 0.404; this represents the unit rate or speed. In this context, it means the height goes up by about 0.404 cm per day. This is an estimated value because the regression line itself is a collection of estimated points.

The y intercept is where the graph crosses the y axis. It always occurs when x = 0. The x refers to the day number. Day 0 is basically the starting day. So the y intercept being -5.18 means the estimated height is -5.18 cm on the starting day. At first glance, it might not seem possible to have a negative height. But simply think of negative heights as below ground level, much like how the negative y values are below the x axis.

In other words, the plant is estimated to start off at about 5.18 cm below ground level. This is a reasonable assumption because the seed is buried into the ground (assuming at this level more or less) and it then grows upward.

Question 3 of 10
In the diagram below, Ed is parallel to Xy. What is the value of x?

Answers

The answer is C because it interior angles and 180-105 =75

The value of y will be 75°. The correct option is C.

What are lines and angles?

Straight lines with little depth or width are present. You will learn about a number of lines, including transversal, intersecting, and perpendicular lines.

A figure called an angle is one in which two rays originate from the same point. In this area, you could also encounter contrasting and related viewpoints.

Given that the two parallel lines are DE and XY. The angle y will be calculated as,

y = 180 - 105

y = 75°

Hence, the measure of the angle y is 75°

To know more about lines and angles follow

https://brainly.com/question/28769265

#SPJ2

please help me immediately​

Answers

Answer:

zero of a polynomial can be defined as the points where the polynomial become 0 as a whole.

An equation formed with variables, exponents and coefficient together with operation and an equal sign is called polynomial equation.

I hope this will help you

please help I will give brainlist​

Answers

Answer:

Your answer will be B

Step-by-step explanation:

Because you find the difference between 14 and 22 or the difference between 22 and 30 or the difference between 30 and 38. and so on then it will be 8

You then say ok 8 times x so for example 8×1 = 8 then you say ok 22 - 8= 22. So it will be 8x+14...

I hope this helped if not tell me please....❤

Answer: Option B (y=8x+14)

Step-by-step explanation:

To solve these kind of question we just simply keep on inserting the values of x and if we get the required value of y that is in the table that equation is our answer, For example lets take Option A,

y = 8x

If we insert x = 0 we get,

y = 8(0)

y=0

but in the table it says we get 14 so option A is incorrect

Like this we keep checking for more values and all the values of x satisfy our equation in Option B like this,

y=8x+14

y=8(0)+14

y=14

y=8x+14

y=8(1)+14

y=8+14

y=22

y=8x+14

y=8(2)+14

y=16+14

y=30

and so on. Now your answer would be Option B.

THE CREDIT GOES TO (IjlalHashmi)

Jaclyn used the slope formula to find the slope of the line through the points given in the table.

A 2-column table with 2 rows. Column 1 is labeled x with entries 0, negative 3. Column 2 is labeled y with entries 4, 0.

1. Point 1: (0, 4); Point 2: (negative 3, 0). 2. Slope = StartFraction y 2 minus y 1 Over x 2 minus x 1 EndFraction = StartFraction 0 minus 4 Over negative 3 minus 0 EndFraction = StartFraction negative 4 Over negative 3 EndFraction. 3. Slope = four-thirds.

In which step did Jaclyn make an error?

Answers

Answer: She didn't make an error.

Step-by-step explanation:

She just didn't I got it right.

Answer:

answer is D

Step-by-step explanation:

Please could someone help me?

Answers

Answer:

C

Step-by-step explanation:  6  23  17  16

15 is the highest value

Answer:

7:00 pm

Step-by-step explanation:

Mode - Result that appears the most

15 people said they preferred the time 7pm. 15 is the largest number out of the given data

So the mode is 7pm

Need help immediately
A sum of money is to be divided among A, B and C in the ratio 2:3:5. The smallest share amounts to $210.
Calculate:
i. The total sum of money to be shared
ii. C’s share
iii. B’s share​

Answers

Answer:

Step-by-step explanation:

If A, B, and C are in the ratio of 2:3:5 and they are each getting some amount of money in that ratio, then to find the total amount of money they split is found in Ax + Bx + Cx and we need to solve for x. What we are told is that the smallest share = 210 and obviously A is the smallest share. Therefore,

2x = 210 and x = 105.

The total sum = 210 + 3(105) + 5(105) so

Total sum = 210 + 315 + 525

Total sum = 1050 and

C's share is 525 and

B's share is 315

The tennis courts are 3 blocks west and 2 blocks south of Miguel’s house. The bike path is 5 blocks east and 2 blocks south of Miguel’s house. How many blocks are the tennis courts from the bike path?

Answers

Answer:

8

Step-by-step explanation:

If Miguel's house is (0,0), then the tennis court is (-3,-2), and the bike path is (5,-2). There fore, we can just count the squares between the tennis court and the bike path, which is 8.

Hope that this helps!

From the coordinate calculation, here tennis courts are 8 blocks far from the bike path.

Given that,
The tennis courts are 3 blocks west and 2 blocks south of Miguel’s house. The bike path is 5 blocks east and 2 blocks south of Miguel’s house. How many blocks are the tennis courts from the bike path is to be detemined.


What is coordinate?

Coordinate, is represented as the values on the x-axis and the y-axis of the graph.

Here on the graph,
Assume that Miguel's house is (0,0),
then accoding to the question the tennis court is (-3,-2), and the bike path is (5,-2).
Now, difference in the position dx = 5 - (-3)
                                                    = 8

Thus, from the coordinate calculation, here tennis courts are 8 blocks far from the bike path.


Learn more about coordinate here:

brainly.com/question/13498438

#SPJ2

A child weighs 44 lbs. The physician orders a medication for 10 mg/Kg/day. What do you give per day? (Use 0.45 for your conversion

Answers

Answer:

198 mg

Step-by-step explanation:

A child weighs 44 lbs.

Step 1

We convert the child's weight to kg

1 Ibs = 0.45kg

44 Ibs = x

Cross Multiply

1 Ibs × x = 44 Ibs × 0.45 kg

x = 44 Ibs × 0.45 kg/1 Ibs

x = 19.8 kg

Step 2

The physician orders a medication for 10 mg/Kg/day.

From the above statement, we know that:

10mg = 1 kg

Hence, what you give per day is calculated as:

1 kg = 10 mg

19.8kg = x

Cross Multiply

1 kg × x = 19.8kg × 10mg

x = 19.8kg × 10mg/1kg

x = 198mg

Therefore, what you will give per day to a child of 19.8kg(44 Ibs) is 198mg per day

Someone help please struggling on this answer thank you so much!

Answers

Answer:

-x+8=6

Step-by-step explanation:

x+5-1(2x)+(-1)(-3)=6

x+5-1-2x+3=6

ans (-x+8=6)

x+8

Other Questions
What should you mainly follow when creating handouts?A. Provide brief but comprehensive information.B. Use different fonts through the document.C. Provide a printout of each slide in the slideshow.D. Provide all information in the form of graphs and images. Calculate the number of kilowatt-hour a 7kW industrial pump used in 15 hours? A:6 and 30 B:8 and 28C:4 and 16 which of the following can lead to an increase of squirrels in a park A.increased emigrationB.increased immigrationC.a large number of predatorsD.high competition for food Penny has jogged a total of 15 miles this week, which is 4 miles further than last week. How many miles (m) did she jog last week? Choose the equation that correctly models this situation.1) 15m = 4m2) m + 4 = 153) 15m + m = 44) m divided by 4 = 15 Find the volume of the prism. 4.8 m10 m7.2 m An 18-year-old college football player comes to your office after sustaining an injury on the field. He states that he planted his foot and had to pivot to catch the ball. He heard a "pop" and his right knee gave way and started to swell. The trainer iced it down. On physical examination you note increased swelling and tenderness over the right knee. He has significant forward excursion when you perform the Lachman test. He also has more movement of the right tibia when you draw it forward when compared with the left. Based on this information what is your most likely diagnosis? What is the name of database of viruses that an antivirus software scan for? the furniture store sells dining room tables and chairs. single chairs for particular table sell for 75.00 each. the cost for four chairs when the customer purchases the entire set is 200. what is the percent savings on the 4-chair set rounded to the nearest whole percent?a.25%b.33%c.50%d.66% Adrcle has a diameter of 4 inches. Which statement about the area and circumference of the circle is true?A comparison of the area and circumference of the circle is not possible because there is not enough informationto find bothThe numerical values of the circumference and area are equalThe numerical value of the circumference is greater than the numeste value of the areaThe numerical value of the circumference is less than the numerical value of the area How does Justice Black support the dissenting opinion?A. By pointing out that people under 18 and 21 are already barredfrom certain rightsB. By arguing that teachers who do not punish their students areunconstitutionalO C. By explaining that citizens who assemble peacefully are threats toour democracyD. By arguing that people who voice their opinions in a place ofworship are free to do soSUBMIT Please answer correctly !!!!! Will mark Brianliest !!!!!!!!!!!!!! 4x-8y less than or equal to -16 Jim asks Bob how he is feeling. Instead of replying verbally to Jim, Bob gives Jim a "thumbs up" to let him know that he is doing well. Sociologists would describe Bob's response to Jim as which of the following: a. Gestureb. Languagec. Signd. Symbol Which statement best explains the ending of the play? According to the Greek Earth-centered model of the universe, apparent retrograde motion Help me with these 20 questions on Law and Government. (This is due tomorrow at 5:00 PM) (I need serious answers) 1. What is government and why does it exist and why do we need it? 2. What ideas influenced the Constitution? 3. What is the importance of the Constitution?4. What is federalism and how does it work?? What are its Strengths & Weaknesses?? 5. How and why do people vote the way they do?? 6. What is the process of political socialization and how does it affect voter behavior?7. How does the American electoral process work?8. How do Presidential and Congressional electoral processes differ?9. What is the role of Congress in government? 10. What is the constitutional role of the legislative branch?? 11. Compare and Contrast the House and the Senate?? 12. What are the limits to legislative power??13. What is the role of the President in American government?14. What is the constitutional role of the Executive Branch?15. How does the executive branch function as an institution??16. What are the limits to executive power??17. What is the role of the courts in American government??18. What is the Constitutional role of the judicial branch?19. What are the limits to judicial power?20. How do civil liberties and civil rights influence civic life? Question 4(Multiple Choice Worth 5 points)(01.05 LC)Prioritization is best defined asO avoiding an unpleasant task by occupying yourself with something less urgentO identifying the most important or urgent tasks over ones that can wait to be tackledswitching back and forth between tasks or tackling more than one task at the same timethe way that you organize and plan how long you spend on specific activitiesQuestion 5(Multiple Choice Worth 5 points)101 05 MC What is idealism and what are the examples of Idealisim.What Idealism is all about? Really struggling with this, can someone please help me out and give me a brief explanation?